Prove the inequality and limitation












5












$begingroup$



Suppose ${a_k}$ ,${b_k}$ and ${xi_k}$ are non negative, and for all $kge 0$ , we have $$a_{k+1}^2le(a_k+b_k)^2-xi_k^2$$
Prove

1.$$sum_{i=1}^{k}xi_i^2le(a_1+sum_{i=0}^kb_i)^2$$

2. If ${b_k}$ also satisfy $sum_{k=0}^{infty}b_k^2lt+infty $, then $$lim_{ktoinfty}frac{1}{k}sum_{i=1}^kxi_i^2=0$$




After doing this transformation:$$xi_k^2le(a_k+b_k)^2-a_{k+1}^2$$
$Rightarrow$ $$sum_{i=1}^{k}xi_k^2le(a_1+b_1-a_2)(a_1+b_1+a_2)+(a_2+b_2-a_3)(a_2+b_2+a_3)+cdots(a_k+b_k-a_{k+1})(a_k+b_k+a_{k+1})$$
then I can't move forward any more.










share|cite|improve this question











$endgroup$












  • $begingroup$
    Is $a_{k}$ decreasing?
    $endgroup$
    – Alex
    Sep 3 '18 at 5:24










  • $begingroup$
    @Alex The condition does not include this.
    $endgroup$
    – Jaqen Chou
    Sep 3 '18 at 5:36










  • $begingroup$
    Could you tell me something about the origin of the problem? I am curious about it!
    $endgroup$
    – Fyhswdsxjj
    Dec 5 '18 at 23:32










  • $begingroup$
    @gcfsjfcus It has been a long time. I just remember it's from some college's entrance examination.
    $endgroup$
    – Jaqen Chou
    Dec 6 '18 at 8:13
















5












$begingroup$



Suppose ${a_k}$ ,${b_k}$ and ${xi_k}$ are non negative, and for all $kge 0$ , we have $$a_{k+1}^2le(a_k+b_k)^2-xi_k^2$$
Prove

1.$$sum_{i=1}^{k}xi_i^2le(a_1+sum_{i=0}^kb_i)^2$$

2. If ${b_k}$ also satisfy $sum_{k=0}^{infty}b_k^2lt+infty $, then $$lim_{ktoinfty}frac{1}{k}sum_{i=1}^kxi_i^2=0$$




After doing this transformation:$$xi_k^2le(a_k+b_k)^2-a_{k+1}^2$$
$Rightarrow$ $$sum_{i=1}^{k}xi_k^2le(a_1+b_1-a_2)(a_1+b_1+a_2)+(a_2+b_2-a_3)(a_2+b_2+a_3)+cdots(a_k+b_k-a_{k+1})(a_k+b_k+a_{k+1})$$
then I can't move forward any more.










share|cite|improve this question











$endgroup$












  • $begingroup$
    Is $a_{k}$ decreasing?
    $endgroup$
    – Alex
    Sep 3 '18 at 5:24










  • $begingroup$
    @Alex The condition does not include this.
    $endgroup$
    – Jaqen Chou
    Sep 3 '18 at 5:36










  • $begingroup$
    Could you tell me something about the origin of the problem? I am curious about it!
    $endgroup$
    – Fyhswdsxjj
    Dec 5 '18 at 23:32










  • $begingroup$
    @gcfsjfcus It has been a long time. I just remember it's from some college's entrance examination.
    $endgroup$
    – Jaqen Chou
    Dec 6 '18 at 8:13














5












5








5


3



$begingroup$



Suppose ${a_k}$ ,${b_k}$ and ${xi_k}$ are non negative, and for all $kge 0$ , we have $$a_{k+1}^2le(a_k+b_k)^2-xi_k^2$$
Prove

1.$$sum_{i=1}^{k}xi_i^2le(a_1+sum_{i=0}^kb_i)^2$$

2. If ${b_k}$ also satisfy $sum_{k=0}^{infty}b_k^2lt+infty $, then $$lim_{ktoinfty}frac{1}{k}sum_{i=1}^kxi_i^2=0$$




After doing this transformation:$$xi_k^2le(a_k+b_k)^2-a_{k+1}^2$$
$Rightarrow$ $$sum_{i=1}^{k}xi_k^2le(a_1+b_1-a_2)(a_1+b_1+a_2)+(a_2+b_2-a_3)(a_2+b_2+a_3)+cdots(a_k+b_k-a_{k+1})(a_k+b_k+a_{k+1})$$
then I can't move forward any more.










share|cite|improve this question











$endgroup$





Suppose ${a_k}$ ,${b_k}$ and ${xi_k}$ are non negative, and for all $kge 0$ , we have $$a_{k+1}^2le(a_k+b_k)^2-xi_k^2$$
Prove

1.$$sum_{i=1}^{k}xi_i^2le(a_1+sum_{i=0}^kb_i)^2$$

2. If ${b_k}$ also satisfy $sum_{k=0}^{infty}b_k^2lt+infty $, then $$lim_{ktoinfty}frac{1}{k}sum_{i=1}^kxi_i^2=0$$




After doing this transformation:$$xi_k^2le(a_k+b_k)^2-a_{k+1}^2$$
$Rightarrow$ $$sum_{i=1}^{k}xi_k^2le(a_1+b_1-a_2)(a_1+b_1+a_2)+(a_2+b_2-a_3)(a_2+b_2+a_3)+cdots(a_k+b_k-a_{k+1})(a_k+b_k+a_{k+1})$$
then I can't move forward any more.







calculus inequality






share|cite|improve this question















share|cite|improve this question













share|cite|improve this question




share|cite|improve this question








edited Sep 1 '18 at 4:20







Jaqen Chou

















asked Aug 31 '18 at 18:18









Jaqen ChouJaqen Chou

422110




422110












  • $begingroup$
    Is $a_{k}$ decreasing?
    $endgroup$
    – Alex
    Sep 3 '18 at 5:24










  • $begingroup$
    @Alex The condition does not include this.
    $endgroup$
    – Jaqen Chou
    Sep 3 '18 at 5:36










  • $begingroup$
    Could you tell me something about the origin of the problem? I am curious about it!
    $endgroup$
    – Fyhswdsxjj
    Dec 5 '18 at 23:32










  • $begingroup$
    @gcfsjfcus It has been a long time. I just remember it's from some college's entrance examination.
    $endgroup$
    – Jaqen Chou
    Dec 6 '18 at 8:13


















  • $begingroup$
    Is $a_{k}$ decreasing?
    $endgroup$
    – Alex
    Sep 3 '18 at 5:24










  • $begingroup$
    @Alex The condition does not include this.
    $endgroup$
    – Jaqen Chou
    Sep 3 '18 at 5:36










  • $begingroup$
    Could you tell me something about the origin of the problem? I am curious about it!
    $endgroup$
    – Fyhswdsxjj
    Dec 5 '18 at 23:32










  • $begingroup$
    @gcfsjfcus It has been a long time. I just remember it's from some college's entrance examination.
    $endgroup$
    – Jaqen Chou
    Dec 6 '18 at 8:13
















$begingroup$
Is $a_{k}$ decreasing?
$endgroup$
– Alex
Sep 3 '18 at 5:24




$begingroup$
Is $a_{k}$ decreasing?
$endgroup$
– Alex
Sep 3 '18 at 5:24












$begingroup$
@Alex The condition does not include this.
$endgroup$
– Jaqen Chou
Sep 3 '18 at 5:36




$begingroup$
@Alex The condition does not include this.
$endgroup$
– Jaqen Chou
Sep 3 '18 at 5:36












$begingroup$
Could you tell me something about the origin of the problem? I am curious about it!
$endgroup$
– Fyhswdsxjj
Dec 5 '18 at 23:32




$begingroup$
Could you tell me something about the origin of the problem? I am curious about it!
$endgroup$
– Fyhswdsxjj
Dec 5 '18 at 23:32












$begingroup$
@gcfsjfcus It has been a long time. I just remember it's from some college's entrance examination.
$endgroup$
– Jaqen Chou
Dec 6 '18 at 8:13




$begingroup$
@gcfsjfcus It has been a long time. I just remember it's from some college's entrance examination.
$endgroup$
– Jaqen Chou
Dec 6 '18 at 8:13










1 Answer
1






active

oldest

votes


















3





+50







$begingroup$

Let us first prove by induction that
$$tag{1} a_1 +sum_{i=0}^k b_i ge a_{k+1}.$$
For $k=0$ this inequality is trivial. Next assume that (1) is already shown for $0 le i le k$, then we have
$$a_1 + sum_{i=0}^{k+1} b_i ge a_{k+1} + b_{k+1} ge a_{k+1} + big(sqrt{a_{k+2}^2 + xi_{k+1}^2} - a_{k+1} big) ge a_{k+2}.$$
Now we are ready to show the stronger inequality
$$tag{2}sum_{i=1}^k xi_i^2 leq big(a_1 + sum_{i=0}^k b_i big)^2 - a_{k+1}^2.$$
Again, we prove this by induction. In fact, $k=1$ is a direct consequence of the initial condition:
$$a_2^2 le (a_1+b_1)^2-xi_1^2 le (a_1+b_0+b_1)^2 - xi_1^2.$$
Assume that (2) is true for $k$, then
begin{align}
big(a_1 + sum_{i=0}^{k+1} b_i big)^2-a_{k+2}^2 &ge sum_{i=1}^k xi_i^2 + 2 big( a_1 +sum_{i=0}^k b_k big) b_{k+1} + b_{k+1}^2 + (a_{k+1}^2 -a_{k+2}^2) \
&ge sum_{i=1}^k xi_i^2 + (2 a_{k+1} b_{k+1} +b_{k+1}^2) + (a_{k+1}^2 -a_{k+2}^2) \
&ge sum_{i=1}^k xi_i^2 + (a_{k+2}^2 +xi_{k+1}^2 - a_{k+1}^2) + (a_{k+1}^2 -a_{k+2}^2) \
&ge sum_{i=1}^{k+1} xi_i^2
end{align}
Note that I have used in (1) the second inequality and the initial condition in the third inequality.



Additionally assuming that $sum_{k=0}^infty b_k^2 < infty$, we can apply the previous result with shifted-indizes in order to conclude (by using $(a+b)^2 le 2 a^2+2b^2$) that
begin{align}
frac{1}{1+k-j} sum_{i=j}^k xi_i^2 &le frac{2}{1+k-j} Big(a_j^2 + (sum_{i=j}^k b_i)^2Big)\
&le frac{2a_j^2}{1+k-j} + 2sum_{i=j}^k b_i^2,
end{align}
where in the last step the Cauchy-Schwarz inequality was used.
Thus
begin{align}
limsup_{k rightarrow infty} frac{1}{k} sum_{i=1}^k xi_i^2 &le limsup_{k rightarrow infty} frac{1}{k} sum_{i=1}^{j-1} xi_i^2 + limsup_{k rightarrow infty} frac{1}{k} sum_{i=j}^k xi_i^2 \
&= limsup_{k rightarrow infty} frac{k+1-j}{k} frac{1}{k+1-j} sum_{i=j}^k xi_i^2 \
&le limsup_{k rightarrow infty} frac{2a_j^2}{1+k-j} + 2sum_{i=j}^k b_i^2 \
&= 2sum_{i=j}^infty b_i^2 rightarrow 0 quad text{for} quad j rightarrow infty
end{align}






share|cite|improve this answer











$endgroup$













  • $begingroup$
    This is impressive! I just have a question about the last inquality. How does $sum_{i=1}^k xi_i^2 le 2(a_1^2+(sum_{i=0}^k b_i)^2)$ come to $sum_{i=1}^k xi_i^2 le 2(a_1^2+sum_{i=0}^k b_i^2)$
    $endgroup$
    – Jaqen Chou
    Sep 4 '18 at 4:24












  • $begingroup$
    My argument was not correct at all. I have added a complete answer.
    $endgroup$
    – p4sch
    Sep 4 '18 at 7:47










  • $begingroup$
    I think the application of Cauchy-Schwarz inequality is not true. We can check it by set $k=j+1$.
    $endgroup$
    – Jaqen Chou
    Sep 4 '18 at 9:00










  • $begingroup$
    By the Cauchy-Schwarz inequality we find $big( sum_{i=j}^k b_i)^2 le big( sum_{i=j}^k 1) big( sum_{i=j}^k b_i^2 big) = (k+1-j) big( sum_{i=j}^k b_i^2 big)$. The factor $(k+1-j)$ cancels with $frac{1}{k+1-j}$.
    $endgroup$
    – p4sch
    Sep 4 '18 at 9:48










  • $begingroup$
    That's right. I have no question any more. Also maybe you could edit $xi_k$ to $xi_i$ in the last formula series.
    $endgroup$
    – Jaqen Chou
    Sep 4 '18 at 13:28











Your Answer





StackExchange.ifUsing("editor", function () {
return StackExchange.using("mathjaxEditing", function () {
StackExchange.MarkdownEditor.creationCallbacks.add(function (editor, postfix) {
StackExchange.mathjaxEditing.prepareWmdForMathJax(editor, postfix, [["$", "$"], ["\\(","\\)"]]);
});
});
}, "mathjax-editing");

StackExchange.ready(function() {
var channelOptions = {
tags: "".split(" "),
id: "69"
};
initTagRenderer("".split(" "), "".split(" "), channelOptions);

StackExchange.using("externalEditor", function() {
// Have to fire editor after snippets, if snippets enabled
if (StackExchange.settings.snippets.snippetsEnabled) {
StackExchange.using("snippets", function() {
createEditor();
});
}
else {
createEditor();
}
});

function createEditor() {
StackExchange.prepareEditor({
heartbeatType: 'answer',
autoActivateHeartbeat: false,
convertImagesToLinks: true,
noModals: true,
showLowRepImageUploadWarning: true,
reputationToPostImages: 10,
bindNavPrevention: true,
postfix: "",
imageUploader: {
brandingHtml: "Powered by u003ca class="icon-imgur-white" href="https://imgur.com/"u003eu003c/au003e",
contentPolicyHtml: "User contributions licensed under u003ca href="https://creativecommons.org/licenses/by-sa/3.0/"u003ecc by-sa 3.0 with attribution requiredu003c/au003e u003ca href="https://stackoverflow.com/legal/content-policy"u003e(content policy)u003c/au003e",
allowUrls: true
},
noCode: true, onDemand: true,
discardSelector: ".discard-answer"
,immediatelyShowMarkdownHelp:true
});


}
});














draft saved

draft discarded


















StackExchange.ready(
function () {
StackExchange.openid.initPostLogin('.new-post-login', 'https%3a%2f%2fmath.stackexchange.com%2fquestions%2f2901014%2fprove-the-inequality-and-limitation%23new-answer', 'question_page');
}
);

Post as a guest















Required, but never shown

























1 Answer
1






active

oldest

votes








1 Answer
1






active

oldest

votes









active

oldest

votes






active

oldest

votes









3





+50







$begingroup$

Let us first prove by induction that
$$tag{1} a_1 +sum_{i=0}^k b_i ge a_{k+1}.$$
For $k=0$ this inequality is trivial. Next assume that (1) is already shown for $0 le i le k$, then we have
$$a_1 + sum_{i=0}^{k+1} b_i ge a_{k+1} + b_{k+1} ge a_{k+1} + big(sqrt{a_{k+2}^2 + xi_{k+1}^2} - a_{k+1} big) ge a_{k+2}.$$
Now we are ready to show the stronger inequality
$$tag{2}sum_{i=1}^k xi_i^2 leq big(a_1 + sum_{i=0}^k b_i big)^2 - a_{k+1}^2.$$
Again, we prove this by induction. In fact, $k=1$ is a direct consequence of the initial condition:
$$a_2^2 le (a_1+b_1)^2-xi_1^2 le (a_1+b_0+b_1)^2 - xi_1^2.$$
Assume that (2) is true for $k$, then
begin{align}
big(a_1 + sum_{i=0}^{k+1} b_i big)^2-a_{k+2}^2 &ge sum_{i=1}^k xi_i^2 + 2 big( a_1 +sum_{i=0}^k b_k big) b_{k+1} + b_{k+1}^2 + (a_{k+1}^2 -a_{k+2}^2) \
&ge sum_{i=1}^k xi_i^2 + (2 a_{k+1} b_{k+1} +b_{k+1}^2) + (a_{k+1}^2 -a_{k+2}^2) \
&ge sum_{i=1}^k xi_i^2 + (a_{k+2}^2 +xi_{k+1}^2 - a_{k+1}^2) + (a_{k+1}^2 -a_{k+2}^2) \
&ge sum_{i=1}^{k+1} xi_i^2
end{align}
Note that I have used in (1) the second inequality and the initial condition in the third inequality.



Additionally assuming that $sum_{k=0}^infty b_k^2 < infty$, we can apply the previous result with shifted-indizes in order to conclude (by using $(a+b)^2 le 2 a^2+2b^2$) that
begin{align}
frac{1}{1+k-j} sum_{i=j}^k xi_i^2 &le frac{2}{1+k-j} Big(a_j^2 + (sum_{i=j}^k b_i)^2Big)\
&le frac{2a_j^2}{1+k-j} + 2sum_{i=j}^k b_i^2,
end{align}
where in the last step the Cauchy-Schwarz inequality was used.
Thus
begin{align}
limsup_{k rightarrow infty} frac{1}{k} sum_{i=1}^k xi_i^2 &le limsup_{k rightarrow infty} frac{1}{k} sum_{i=1}^{j-1} xi_i^2 + limsup_{k rightarrow infty} frac{1}{k} sum_{i=j}^k xi_i^2 \
&= limsup_{k rightarrow infty} frac{k+1-j}{k} frac{1}{k+1-j} sum_{i=j}^k xi_i^2 \
&le limsup_{k rightarrow infty} frac{2a_j^2}{1+k-j} + 2sum_{i=j}^k b_i^2 \
&= 2sum_{i=j}^infty b_i^2 rightarrow 0 quad text{for} quad j rightarrow infty
end{align}






share|cite|improve this answer











$endgroup$













  • $begingroup$
    This is impressive! I just have a question about the last inquality. How does $sum_{i=1}^k xi_i^2 le 2(a_1^2+(sum_{i=0}^k b_i)^2)$ come to $sum_{i=1}^k xi_i^2 le 2(a_1^2+sum_{i=0}^k b_i^2)$
    $endgroup$
    – Jaqen Chou
    Sep 4 '18 at 4:24












  • $begingroup$
    My argument was not correct at all. I have added a complete answer.
    $endgroup$
    – p4sch
    Sep 4 '18 at 7:47










  • $begingroup$
    I think the application of Cauchy-Schwarz inequality is not true. We can check it by set $k=j+1$.
    $endgroup$
    – Jaqen Chou
    Sep 4 '18 at 9:00










  • $begingroup$
    By the Cauchy-Schwarz inequality we find $big( sum_{i=j}^k b_i)^2 le big( sum_{i=j}^k 1) big( sum_{i=j}^k b_i^2 big) = (k+1-j) big( sum_{i=j}^k b_i^2 big)$. The factor $(k+1-j)$ cancels with $frac{1}{k+1-j}$.
    $endgroup$
    – p4sch
    Sep 4 '18 at 9:48










  • $begingroup$
    That's right. I have no question any more. Also maybe you could edit $xi_k$ to $xi_i$ in the last formula series.
    $endgroup$
    – Jaqen Chou
    Sep 4 '18 at 13:28
















3





+50







$begingroup$

Let us first prove by induction that
$$tag{1} a_1 +sum_{i=0}^k b_i ge a_{k+1}.$$
For $k=0$ this inequality is trivial. Next assume that (1) is already shown for $0 le i le k$, then we have
$$a_1 + sum_{i=0}^{k+1} b_i ge a_{k+1} + b_{k+1} ge a_{k+1} + big(sqrt{a_{k+2}^2 + xi_{k+1}^2} - a_{k+1} big) ge a_{k+2}.$$
Now we are ready to show the stronger inequality
$$tag{2}sum_{i=1}^k xi_i^2 leq big(a_1 + sum_{i=0}^k b_i big)^2 - a_{k+1}^2.$$
Again, we prove this by induction. In fact, $k=1$ is a direct consequence of the initial condition:
$$a_2^2 le (a_1+b_1)^2-xi_1^2 le (a_1+b_0+b_1)^2 - xi_1^2.$$
Assume that (2) is true for $k$, then
begin{align}
big(a_1 + sum_{i=0}^{k+1} b_i big)^2-a_{k+2}^2 &ge sum_{i=1}^k xi_i^2 + 2 big( a_1 +sum_{i=0}^k b_k big) b_{k+1} + b_{k+1}^2 + (a_{k+1}^2 -a_{k+2}^2) \
&ge sum_{i=1}^k xi_i^2 + (2 a_{k+1} b_{k+1} +b_{k+1}^2) + (a_{k+1}^2 -a_{k+2}^2) \
&ge sum_{i=1}^k xi_i^2 + (a_{k+2}^2 +xi_{k+1}^2 - a_{k+1}^2) + (a_{k+1}^2 -a_{k+2}^2) \
&ge sum_{i=1}^{k+1} xi_i^2
end{align}
Note that I have used in (1) the second inequality and the initial condition in the third inequality.



Additionally assuming that $sum_{k=0}^infty b_k^2 < infty$, we can apply the previous result with shifted-indizes in order to conclude (by using $(a+b)^2 le 2 a^2+2b^2$) that
begin{align}
frac{1}{1+k-j} sum_{i=j}^k xi_i^2 &le frac{2}{1+k-j} Big(a_j^2 + (sum_{i=j}^k b_i)^2Big)\
&le frac{2a_j^2}{1+k-j} + 2sum_{i=j}^k b_i^2,
end{align}
where in the last step the Cauchy-Schwarz inequality was used.
Thus
begin{align}
limsup_{k rightarrow infty} frac{1}{k} sum_{i=1}^k xi_i^2 &le limsup_{k rightarrow infty} frac{1}{k} sum_{i=1}^{j-1} xi_i^2 + limsup_{k rightarrow infty} frac{1}{k} sum_{i=j}^k xi_i^2 \
&= limsup_{k rightarrow infty} frac{k+1-j}{k} frac{1}{k+1-j} sum_{i=j}^k xi_i^2 \
&le limsup_{k rightarrow infty} frac{2a_j^2}{1+k-j} + 2sum_{i=j}^k b_i^2 \
&= 2sum_{i=j}^infty b_i^2 rightarrow 0 quad text{for} quad j rightarrow infty
end{align}






share|cite|improve this answer











$endgroup$













  • $begingroup$
    This is impressive! I just have a question about the last inquality. How does $sum_{i=1}^k xi_i^2 le 2(a_1^2+(sum_{i=0}^k b_i)^2)$ come to $sum_{i=1}^k xi_i^2 le 2(a_1^2+sum_{i=0}^k b_i^2)$
    $endgroup$
    – Jaqen Chou
    Sep 4 '18 at 4:24












  • $begingroup$
    My argument was not correct at all. I have added a complete answer.
    $endgroup$
    – p4sch
    Sep 4 '18 at 7:47










  • $begingroup$
    I think the application of Cauchy-Schwarz inequality is not true. We can check it by set $k=j+1$.
    $endgroup$
    – Jaqen Chou
    Sep 4 '18 at 9:00










  • $begingroup$
    By the Cauchy-Schwarz inequality we find $big( sum_{i=j}^k b_i)^2 le big( sum_{i=j}^k 1) big( sum_{i=j}^k b_i^2 big) = (k+1-j) big( sum_{i=j}^k b_i^2 big)$. The factor $(k+1-j)$ cancels with $frac{1}{k+1-j}$.
    $endgroup$
    – p4sch
    Sep 4 '18 at 9:48










  • $begingroup$
    That's right. I have no question any more. Also maybe you could edit $xi_k$ to $xi_i$ in the last formula series.
    $endgroup$
    – Jaqen Chou
    Sep 4 '18 at 13:28














3





+50







3





+50



3




+50



$begingroup$

Let us first prove by induction that
$$tag{1} a_1 +sum_{i=0}^k b_i ge a_{k+1}.$$
For $k=0$ this inequality is trivial. Next assume that (1) is already shown for $0 le i le k$, then we have
$$a_1 + sum_{i=0}^{k+1} b_i ge a_{k+1} + b_{k+1} ge a_{k+1} + big(sqrt{a_{k+2}^2 + xi_{k+1}^2} - a_{k+1} big) ge a_{k+2}.$$
Now we are ready to show the stronger inequality
$$tag{2}sum_{i=1}^k xi_i^2 leq big(a_1 + sum_{i=0}^k b_i big)^2 - a_{k+1}^2.$$
Again, we prove this by induction. In fact, $k=1$ is a direct consequence of the initial condition:
$$a_2^2 le (a_1+b_1)^2-xi_1^2 le (a_1+b_0+b_1)^2 - xi_1^2.$$
Assume that (2) is true for $k$, then
begin{align}
big(a_1 + sum_{i=0}^{k+1} b_i big)^2-a_{k+2}^2 &ge sum_{i=1}^k xi_i^2 + 2 big( a_1 +sum_{i=0}^k b_k big) b_{k+1} + b_{k+1}^2 + (a_{k+1}^2 -a_{k+2}^2) \
&ge sum_{i=1}^k xi_i^2 + (2 a_{k+1} b_{k+1} +b_{k+1}^2) + (a_{k+1}^2 -a_{k+2}^2) \
&ge sum_{i=1}^k xi_i^2 + (a_{k+2}^2 +xi_{k+1}^2 - a_{k+1}^2) + (a_{k+1}^2 -a_{k+2}^2) \
&ge sum_{i=1}^{k+1} xi_i^2
end{align}
Note that I have used in (1) the second inequality and the initial condition in the third inequality.



Additionally assuming that $sum_{k=0}^infty b_k^2 < infty$, we can apply the previous result with shifted-indizes in order to conclude (by using $(a+b)^2 le 2 a^2+2b^2$) that
begin{align}
frac{1}{1+k-j} sum_{i=j}^k xi_i^2 &le frac{2}{1+k-j} Big(a_j^2 + (sum_{i=j}^k b_i)^2Big)\
&le frac{2a_j^2}{1+k-j} + 2sum_{i=j}^k b_i^2,
end{align}
where in the last step the Cauchy-Schwarz inequality was used.
Thus
begin{align}
limsup_{k rightarrow infty} frac{1}{k} sum_{i=1}^k xi_i^2 &le limsup_{k rightarrow infty} frac{1}{k} sum_{i=1}^{j-1} xi_i^2 + limsup_{k rightarrow infty} frac{1}{k} sum_{i=j}^k xi_i^2 \
&= limsup_{k rightarrow infty} frac{k+1-j}{k} frac{1}{k+1-j} sum_{i=j}^k xi_i^2 \
&le limsup_{k rightarrow infty} frac{2a_j^2}{1+k-j} + 2sum_{i=j}^k b_i^2 \
&= 2sum_{i=j}^infty b_i^2 rightarrow 0 quad text{for} quad j rightarrow infty
end{align}






share|cite|improve this answer











$endgroup$



Let us first prove by induction that
$$tag{1} a_1 +sum_{i=0}^k b_i ge a_{k+1}.$$
For $k=0$ this inequality is trivial. Next assume that (1) is already shown for $0 le i le k$, then we have
$$a_1 + sum_{i=0}^{k+1} b_i ge a_{k+1} + b_{k+1} ge a_{k+1} + big(sqrt{a_{k+2}^2 + xi_{k+1}^2} - a_{k+1} big) ge a_{k+2}.$$
Now we are ready to show the stronger inequality
$$tag{2}sum_{i=1}^k xi_i^2 leq big(a_1 + sum_{i=0}^k b_i big)^2 - a_{k+1}^2.$$
Again, we prove this by induction. In fact, $k=1$ is a direct consequence of the initial condition:
$$a_2^2 le (a_1+b_1)^2-xi_1^2 le (a_1+b_0+b_1)^2 - xi_1^2.$$
Assume that (2) is true for $k$, then
begin{align}
big(a_1 + sum_{i=0}^{k+1} b_i big)^2-a_{k+2}^2 &ge sum_{i=1}^k xi_i^2 + 2 big( a_1 +sum_{i=0}^k b_k big) b_{k+1} + b_{k+1}^2 + (a_{k+1}^2 -a_{k+2}^2) \
&ge sum_{i=1}^k xi_i^2 + (2 a_{k+1} b_{k+1} +b_{k+1}^2) + (a_{k+1}^2 -a_{k+2}^2) \
&ge sum_{i=1}^k xi_i^2 + (a_{k+2}^2 +xi_{k+1}^2 - a_{k+1}^2) + (a_{k+1}^2 -a_{k+2}^2) \
&ge sum_{i=1}^{k+1} xi_i^2
end{align}
Note that I have used in (1) the second inequality and the initial condition in the third inequality.



Additionally assuming that $sum_{k=0}^infty b_k^2 < infty$, we can apply the previous result with shifted-indizes in order to conclude (by using $(a+b)^2 le 2 a^2+2b^2$) that
begin{align}
frac{1}{1+k-j} sum_{i=j}^k xi_i^2 &le frac{2}{1+k-j} Big(a_j^2 + (sum_{i=j}^k b_i)^2Big)\
&le frac{2a_j^2}{1+k-j} + 2sum_{i=j}^k b_i^2,
end{align}
where in the last step the Cauchy-Schwarz inequality was used.
Thus
begin{align}
limsup_{k rightarrow infty} frac{1}{k} sum_{i=1}^k xi_i^2 &le limsup_{k rightarrow infty} frac{1}{k} sum_{i=1}^{j-1} xi_i^2 + limsup_{k rightarrow infty} frac{1}{k} sum_{i=j}^k xi_i^2 \
&= limsup_{k rightarrow infty} frac{k+1-j}{k} frac{1}{k+1-j} sum_{i=j}^k xi_i^2 \
&le limsup_{k rightarrow infty} frac{2a_j^2}{1+k-j} + 2sum_{i=j}^k b_i^2 \
&= 2sum_{i=j}^infty b_i^2 rightarrow 0 quad text{for} quad j rightarrow infty
end{align}







share|cite|improve this answer














share|cite|improve this answer



share|cite|improve this answer








edited Sep 4 '18 at 13:37

























answered Sep 3 '18 at 13:01









p4schp4sch

4,995217




4,995217












  • $begingroup$
    This is impressive! I just have a question about the last inquality. How does $sum_{i=1}^k xi_i^2 le 2(a_1^2+(sum_{i=0}^k b_i)^2)$ come to $sum_{i=1}^k xi_i^2 le 2(a_1^2+sum_{i=0}^k b_i^2)$
    $endgroup$
    – Jaqen Chou
    Sep 4 '18 at 4:24












  • $begingroup$
    My argument was not correct at all. I have added a complete answer.
    $endgroup$
    – p4sch
    Sep 4 '18 at 7:47










  • $begingroup$
    I think the application of Cauchy-Schwarz inequality is not true. We can check it by set $k=j+1$.
    $endgroup$
    – Jaqen Chou
    Sep 4 '18 at 9:00










  • $begingroup$
    By the Cauchy-Schwarz inequality we find $big( sum_{i=j}^k b_i)^2 le big( sum_{i=j}^k 1) big( sum_{i=j}^k b_i^2 big) = (k+1-j) big( sum_{i=j}^k b_i^2 big)$. The factor $(k+1-j)$ cancels with $frac{1}{k+1-j}$.
    $endgroup$
    – p4sch
    Sep 4 '18 at 9:48










  • $begingroup$
    That's right. I have no question any more. Also maybe you could edit $xi_k$ to $xi_i$ in the last formula series.
    $endgroup$
    – Jaqen Chou
    Sep 4 '18 at 13:28


















  • $begingroup$
    This is impressive! I just have a question about the last inquality. How does $sum_{i=1}^k xi_i^2 le 2(a_1^2+(sum_{i=0}^k b_i)^2)$ come to $sum_{i=1}^k xi_i^2 le 2(a_1^2+sum_{i=0}^k b_i^2)$
    $endgroup$
    – Jaqen Chou
    Sep 4 '18 at 4:24












  • $begingroup$
    My argument was not correct at all. I have added a complete answer.
    $endgroup$
    – p4sch
    Sep 4 '18 at 7:47










  • $begingroup$
    I think the application of Cauchy-Schwarz inequality is not true. We can check it by set $k=j+1$.
    $endgroup$
    – Jaqen Chou
    Sep 4 '18 at 9:00










  • $begingroup$
    By the Cauchy-Schwarz inequality we find $big( sum_{i=j}^k b_i)^2 le big( sum_{i=j}^k 1) big( sum_{i=j}^k b_i^2 big) = (k+1-j) big( sum_{i=j}^k b_i^2 big)$. The factor $(k+1-j)$ cancels with $frac{1}{k+1-j}$.
    $endgroup$
    – p4sch
    Sep 4 '18 at 9:48










  • $begingroup$
    That's right. I have no question any more. Also maybe you could edit $xi_k$ to $xi_i$ in the last formula series.
    $endgroup$
    – Jaqen Chou
    Sep 4 '18 at 13:28
















$begingroup$
This is impressive! I just have a question about the last inquality. How does $sum_{i=1}^k xi_i^2 le 2(a_1^2+(sum_{i=0}^k b_i)^2)$ come to $sum_{i=1}^k xi_i^2 le 2(a_1^2+sum_{i=0}^k b_i^2)$
$endgroup$
– Jaqen Chou
Sep 4 '18 at 4:24






$begingroup$
This is impressive! I just have a question about the last inquality. How does $sum_{i=1}^k xi_i^2 le 2(a_1^2+(sum_{i=0}^k b_i)^2)$ come to $sum_{i=1}^k xi_i^2 le 2(a_1^2+sum_{i=0}^k b_i^2)$
$endgroup$
– Jaqen Chou
Sep 4 '18 at 4:24














$begingroup$
My argument was not correct at all. I have added a complete answer.
$endgroup$
– p4sch
Sep 4 '18 at 7:47




$begingroup$
My argument was not correct at all. I have added a complete answer.
$endgroup$
– p4sch
Sep 4 '18 at 7:47












$begingroup$
I think the application of Cauchy-Schwarz inequality is not true. We can check it by set $k=j+1$.
$endgroup$
– Jaqen Chou
Sep 4 '18 at 9:00




$begingroup$
I think the application of Cauchy-Schwarz inequality is not true. We can check it by set $k=j+1$.
$endgroup$
– Jaqen Chou
Sep 4 '18 at 9:00












$begingroup$
By the Cauchy-Schwarz inequality we find $big( sum_{i=j}^k b_i)^2 le big( sum_{i=j}^k 1) big( sum_{i=j}^k b_i^2 big) = (k+1-j) big( sum_{i=j}^k b_i^2 big)$. The factor $(k+1-j)$ cancels with $frac{1}{k+1-j}$.
$endgroup$
– p4sch
Sep 4 '18 at 9:48




$begingroup$
By the Cauchy-Schwarz inequality we find $big( sum_{i=j}^k b_i)^2 le big( sum_{i=j}^k 1) big( sum_{i=j}^k b_i^2 big) = (k+1-j) big( sum_{i=j}^k b_i^2 big)$. The factor $(k+1-j)$ cancels with $frac{1}{k+1-j}$.
$endgroup$
– p4sch
Sep 4 '18 at 9:48












$begingroup$
That's right. I have no question any more. Also maybe you could edit $xi_k$ to $xi_i$ in the last formula series.
$endgroup$
– Jaqen Chou
Sep 4 '18 at 13:28




$begingroup$
That's right. I have no question any more. Also maybe you could edit $xi_k$ to $xi_i$ in the last formula series.
$endgroup$
– Jaqen Chou
Sep 4 '18 at 13:28


















draft saved

draft discarded




















































Thanks for contributing an answer to Mathematics Stack Exchange!


  • Please be sure to answer the question. Provide details and share your research!

But avoid



  • Asking for help, clarification, or responding to other answers.

  • Making statements based on opinion; back them up with references or personal experience.


Use MathJax to format equations. MathJax reference.


To learn more, see our tips on writing great answers.




draft saved


draft discarded














StackExchange.ready(
function () {
StackExchange.openid.initPostLogin('.new-post-login', 'https%3a%2f%2fmath.stackexchange.com%2fquestions%2f2901014%2fprove-the-inequality-and-limitation%23new-answer', 'question_page');
}
);

Post as a guest















Required, but never shown





















































Required, but never shown














Required, but never shown












Required, but never shown







Required, but never shown

































Required, but never shown














Required, but never shown












Required, but never shown







Required, but never shown







Popular posts from this blog

Quarter-circle Tiles

build a pushdown automaton that recognizes the reverse language of a given pushdown automaton?

Mont Emei